Tải bản đầy đủ (.docx) (9 trang)

Cau5TrongdethiDHlay10diem

Bạn đang xem bản rút gọn của tài liệu. Xem và tải ngay bản đầy đủ của tài liệu tại đây (220.13 KB, 9 trang )

<span class='text_page_counter'>(1)</span><div class='page_container' data-page=1>

CÂU V TRONG CÁC ĐỀ THI ĐẠI HỌC


<b>Câu V </b>(1 điểm): Cho x,y,z là ba số thực dương có tổng bằng 3.Tìm giá trị nhỏ nhất của biểu thức
<i>P</i>3(<i>x</i>2<i>y</i>2<i>z</i>2) 2 <i>xyz</i>.<b> Ta có: </b>


<b> </b>




2


3 ( ) 2( ) 2


3 9 2( ) 2


27 6 ( ) 2 ( 3)


<i>P</i> <i>x y z</i> <i>xy yz zx</i> <i>xyz</i>


<i>xy yz zx</i> <i>xyz</i>
<i>x y z</i> <i>yz x</i>


 


 <sub></sub>      <sub></sub> 


    


    


2



3 2


( )


27 6 (3 ) ( 3)


2
1


( 15 27 27)


2


<i>y z</i>


<i>x</i> <i>x</i> <i>x</i>


<i>x</i> <i>x</i> <i>x</i>




    


    


Xét hàm số <i>f x</i>( )<i>x</i>315<i>x</i>2 27<i>x</i>27 , với 0<x<3
,<sub>( )</sub> <sub>3</sub> 2 <sub>30</sub> <sub>27 0</sub> 1


9


<i>x</i>


<i>f x</i> <i>x</i> <i>x</i>


<i>x</i>



   <sub>  </sub>





Từ bảng biến thiên suy ra MinP=7  <i>x</i>  <i>y z</i> 1.<b> </b>


<b>Câu V </b><i><b>(1 điểm)</b></i> Cho a, b, c, d là các số dương. Chứng minh rằng:


<i>abcd</i>


<i>a</i>4 <i>b</i>4 <i>c</i>4 <i>abcd b</i>4 <i>c</i>4 <i>d</i>4 <i>abcd c</i>4 <i>d</i>4 <i>a</i>4 <i>abcd d</i>4 <i>a</i>4 <i>b</i>4 <i>abcd</i>


1 1 1 1 1


   


           


<i>a</i>4<i>b</i>42<i>a b (1); b</i>2 2 4<i>c</i>42<i>b c (2); c</i>2 2 4 <i>a</i>42<i>c a (3)</i>2 2


 <i>a</i>4<i>b</i>4<i>c</i>4<i>abc a b c</i>(   ) <i>a</i>4<i>b</i>4<i>c</i>4<i>abcd abc a b c d</i> (    )


<i> (4)</i>


<i>abc a b c d</i>
<i>a</i>4 <i>b</i>4 <i>c</i>4 <i>abcd</i>


1 1


( )


 


  


   <sub></sub><sub> đpcm.</sub>


<b>Câu V</b> (1đ): Biết ( ; )<i>x y</i> là nghiệm của bất phương trình:5<i>x</i>25<i>y</i>2 5<i>x</i>15<i>y</i> 8 0<sub>. Hãy tìm giá</sub>


trị lớn nhất của biểu thức <i>F x</i> 3<i>y</i><sub>.</sub>


<b> </b> Thay <i>x=F −</i>3<i>y</i> vào bpt ta được: 50<i>y</i>2 30<i>Fy</i>5<i>F</i>2 5<i>F</i> 8 0


Vì bpt ln tồn tại <i>y</i> nên <i>Δ<sub>y</sub>≥</i>0 <i>⇔</i> <i>−</i>25<i>F</i>2


+250<i>F −</i>400<i>≥</i>0 <i>⇔</i> 2≤ F ≤8
Vậy GTLN của <i>F</i>=x+3<i>y</i> là 8.


<b>Câu V</b> (1.0 điểm). Cho x, y, z là các số dương. Chứng minh:


<i>x</i> <i>y</i> <i>z</i> <i>xy</i> <i>yz</i> <i>zx</i>



3 2 4  3 5


Áp dụng BĐT Cô–si:



1 3 5


; 3 ; 5


2 <i>x y</i>  <i>xy</i> 2 <i>y z</i>  <i>xy</i> 2 <i>z x</i>  <i>xy</i> <sub></sub><sub> đpcm</sub>


<b>Câu VIIa (</b>1 điểm<b>): </b>Cho ba số thực dương a, b, c thỏa mãn abc = 1. Chứng minh rằng:


3 3 3


4 4 4


3
(1 )(1 ) (1  )(1 ) (1  )(1 )


<i>a</i> <i>b</i> <i>c</i>


<i>b</i> <i>c</i> <i>c</i> <i>a</i> <i>a</i> <i>b</i> <sub>(4)</sub>


<b>Câu VII.a:</b> Áp dụng BĐT Cô–si ta có:


3 <sub>1</sub> <sub>1</sub> <sub>3</sub> 3 <sub>1</sub> <sub>1</sub> <sub>3</sub> 3 <sub>1</sub> <sub>1</sub> <sub>3</sub>


; ;


(1 )(1 ) 8 8 4 (1 )(1 ) 8 8 4 (1 )(1 ) 8 8 4



     


        


     


<i>a</i> <i>b</i> <i>c</i> <i>a</i> <i>b</i> <i>c</i> <i>a</i> <i>b</i> <i>c</i> <i>a</i> <i>b</i> <i>c</i>


<i>b</i> <i>c</i> <i>c</i> <i>a</i> <i>a</i> <i>b</i>




3 3 3 <sub>3 3</sub>3 <sub>3</sub> <sub>3</sub>


(1 )(1 ) (1 )(1 ) (1 )(1 ) 2 4 2 4 4


 


      


     


<i>a</i> <i>b</i> <i>c</i> <i>a b c</i> <i>abc</i>


<i>b</i> <i>c</i> <i>c</i> <i>a</i> <i>a</i> <i>b</i>


Dấu "=" xảy ra  a = b = c = 1.


<b>Câu V (1 điểm) </b>Cho x,y là các số thực thỏa mãn điều kiện x2<sub>+xy+y</sub>2



 3 .Chứng minh


rằng:


–4 3 3– <i>x</i>2– –<i>xy</i> 3<i>y</i>24 3 3


</div>
<span class='text_page_counter'>(2)</span><div class='page_container' data-page=2>

 Nếu y = 0 thì B = <i>x</i>2  0  B  3
 Nếu y  0 thì đặt t =


<i>x</i>


<i>y</i><sub> ta được B = A. </sub>


2 2 2


2 2 2


3 <sub>.</sub> 3


1


   




   


<i>x</i> <i>xy</i> <i>y</i> <i><sub>A</sub>t</i> <i>t</i>



<i>x</i> <i>xy y</i> <i>t</i> <i>t</i>


Xét phương trình:


2
2


3
1


 

 


<i>t</i> <i>t</i> <i><sub>m</sub></i>


<i>t</i> <i>t</i> <sub></sub><sub> (m–1)t</sub>2<sub> + (m+1)t + m + 3 = 0 (1)</sub>


(1) có nghiệm  m = 1 hoặc  = (m+1)2 – 4(m–1)(m+3)  0


3 4 3
3


 


 m 


3 4 3
3



 


Vì 0  A  3 nên –3–4 3 B  –3+4 3


<b>Câu V (2.0 điểm).</b> Cho a, b, c là ba số dương. Chứng minh rằng:


3 2 3 2 3 2 6


 


  


     


<i>ab</i> <i>bc</i> <i>ca</i> <i>a b c</i>


<i>a</i> <i>b</i> <i>c b</i> <i>c</i> <i>a c</i> <i>a</i> <i>b</i> <b><sub>Câu V: </sub></b><sub>Sử dụng BĐT:</sub>


1 1 1 1 1 1 9


(   )<sub></sub>   <sub></sub> 9   
 


 


<i>x y z</i>


<i>x</i> <i>y</i> <i>z</i> <i>x</i> <i>y</i> <i>z</i> <i>x y z</i>



Ta có:


1 1 1 1 1


.


3 2 ( ) ( ) 2 9 2


 


  <sub></sub>   <sub></sub>


         


<i>ab</i>


<i>ab</i> <i>ab</i>


<i>a</i> <i>b</i> <i>c</i> <i>a c</i> <i>b c</i> <i>b</i> <i>a c b c</i> <i>b</i>


Tương tự đối với 2 biểu thức còn lại. Sau đó cộng vế với vế ta được:


1


3 2 3 2 3 2 9 2 6


      


 



   <sub></sub>    <sub></sub>


          


<i>ab</i> <i>bc</i> <i>ca</i> <i>a b c bc ca ca ab ab bc</i> <i>a b c</i>


<i>a</i> <i>b</i> <i>c b</i> <i>c</i> <i>a c</i> <i>a</i> <i>b</i> <i>a b</i> <i>b c</i> <i>a c</i>


<b>Câu V</b>.(1 điểm) Cho<i>x y</i>, là các số thực thỏa mãn <i>x</i>2<i>y</i>2 <i>xy</i>1.Tìm GTLN, GTNN của
6 6 <sub>2</sub> 2 2


<i>F x</i> <i>y</i>  <i>x y</i>  <i>xy</i>


Giải: Cho<i>x y</i>, là các số thực thỏa mãn <i>x</i>2 <i>y</i>2  <i>xy</i>1.Tìm GTLN, GTNN của <i>F x</i> 6 <i>y</i>62<i>x y</i>2 2<i>xy</i>.


Ta có



3


2 2 <sub>3</sub> 2 2 2 2 <sub>2</sub> 2 2
<i>F</i>  <i>x</i> <i>y</i>  <i>x y x</i> <i>y</i>  <i>x y</i>  <i>xy</i>


=



3 2


2 <i>xy</i> 2 <i>xy</i> 2<i>xy</i> 1


   



Đặt <i>xy t</i> . Ta có <i>f t</i>

 

2<i>t</i>3 2<i>t</i>22<i>t</i>1

2


2 2 <sub>1</sub> <sub>3</sub> <sub>1</sub>


<i>x</i> <i>y</i>  <i>xy</i>  <i>x y</i>  <i>xy</i>


1
3
<i>xy</i> 


 


2


2 2 <sub>1</sub> <sub>1</sub>


<i>x</i> <i>y</i>  <i>xy</i>  <i>x y</i> <i>xy</i> <sub></sub> <i><sub>xy</sub></i><sub></sub><sub>1</sub>


suy ra


1
;1
3


<i>t</i> <sub></sub> <sub></sub>




<i><b>Câu V</b>(1 điểm).</i> Xét ba số thực không âm a, b, c thỏa mÃn a2009<sub> + b</sub>2009<sub> + c</sub>2009<sub> = 3. Tìm giá trÞ lín nhÊt cđa</sub>



biĨu thøc P = a4<sub> + b</sub>4<sub> + c</sub>4


áp dụng bất đẳng thức Cô si cho 2005 số 1 và 4 số a2009<sub> ta có</sub>
1+1+.. .+1



2005


+<i>a</i>2009+<i>a</i>2009+<i>a</i>2009+<i>a</i>2009<i>≥</i>2009.2009

<i>a</i>2009.<i>a</i>2009.<i>a</i>2009.a2009=2009 .<i>a</i>4(1)
T¬ng tù ta cã


1+1+.. .+1



2005


+b2009+<i>b</i>2009+<i>b</i>2009+b2009<i>≥</i>2009 .2009

<i>b</i>2009.<i>b</i>2009.<i>b</i>2009.<i>b</i>2009=2009.<i>b</i>4(2)
1+1+.. .+1



2005


+<i>c</i>2009+<i>c</i>2009+<i>c</i>2009+<i>c</i>2009<i>≥</i>2009.2009

<i>c</i>2009.c2009.<i>c</i>2009.<i>c</i>2009=2009 .<i>c</i>4(3)
Cộng theo vế (1), (2), (3) ta c


6015+4(<i>a</i>2009+<i>b</i>2009+<i>c</i>2009)<i></i>2009(<i>a</i>4+<i>b</i>4+<i>c</i>4)
<i></i>6027<i></i>2009(<i>a</i>4+<i>b</i>4+<i>c</i>4)


T ú suy ra <i><sub>P=</sub><sub>a</sub></i>4



+b4+c4<i></i>3


Mặt khác tại a = b = c = 1 thì P = 3 nên giá trị lớn nhất của P = 3.


</div>
<span class='text_page_counter'>(3)</span><div class='page_container' data-page=3>



9 9 9 9 9 9


6 3 3 6 6 3 3 6 6 3 3 6


<i>x</i> <i>y</i> <i>y</i> <i>z</i> <i>z</i> <i>x</i>


<i>P</i>


<i>x</i> <i>x y</i> <i>y</i> <i>y</i> <i>y z</i> <i>z</i> <i>z</i> <i>z x</i> <i>x</i>


  


  


     


Có x, y, z >0, Đặt : a = x3<sub> , b = y</sub>3<sub>, c = z</sub>3 <sub>(a, b, c >0 ; abc=1)đc :</sub>


3 3 3 3 3 3


2 2 2 2 2 2


<i>a</i> <i>b</i> <i>b</i> <i>c</i> <i>c</i> <i>a</i>



<i>P</i>


<i>a</i> <i>ab b</i> <i>b</i> <i>bc c</i> <i>c</i> <i>ca a</i>


  


  


     


3 3 2 2


2 2 ( ) 2 2


<i>a</i> <i>b</i> <i>a</i> <i>ab b</i>


<i>a b</i>


<i>a</i> <i>ab b</i> <i>a</i> <i>ab b</i>


  


 


    <sub> mà </sub>


2 2


2 2



1
3
<i>a</i> <i>ab b</i>
<i>a</i> <i>ab b</i>


 




  <sub>(Biến đổi tương đương)</sub>


2 2


2 2


1


( ) ( )


3
<i>a</i> <i>ab b</i>


<i>a b</i> <i>a b</i>


<i>a</i> <i>ab b</i>


 


   



 


Tương tự:


3 3 3 3


2 2 2 2


1 1


( ); ( )


3 3


<i>b</i> <i>c</i> <i>c</i> <i>a</i>


<i>b c</i> <i>c a</i>


<i>b</i> <i>bc c</i> <i>c</i> <i>ca a</i>


 


   


   


=>


3


2


( ) 2. 2


3


<i>P</i> <i>a b c</i>   <i>abc</i> 


(BĐT Côsi)
=> P2,<i>P</i>2 khi a = b = c = 1 x = y = z = 1
Vậy: minP = 2 khi x = y =z =1


Câu IV. (1.0 điểm)


Cho x, y, z là các số thực dương lớn hơn 1 và thoả mãn điều kiện xy + yz + zx  2xyz


Tìm giá trị lớn nhất của biểu thức A = (x - 1)(y - 1)(z - 1).
Ta có


1 1 1


2 2


<i>xy yz xz</i> <i>xyz</i>


<i>x</i> <i>y</i> <i>z</i>


      


nên



1 1 1 1 1 ( 1)( 1)


1 1 <i>y</i> <i>z</i> 2 <i>y</i> <i>z</i> (1)


<i>x</i> <i>y</i> <i>z</i> <i>y</i> <i>z</i> <i>yz</i>


   


      


Tương tự ta có


1 1 1 1 1 ( 1)( 1)


1 1 <i>x</i> <i>z</i> 2 <i>x</i> <i>z</i> (2)


<i>y</i> <i>x</i> <i>z</i> <i>x</i> <i>z</i> <i>xz</i>


   


      


1 1 1 1 1 ( 1)( 1)


1 1 <i>x</i> <i>y</i> 2 <i>x</i> <i>y</i> (3)


<i>y</i> <i>x</i> <i>y</i> <i>x</i> <i>y</i> <i>xy</i>


   



      


Nhân vế với vế của (1), (2), (3) ta được


1
( 1)( 1)( 1)


8
<i>x</i> <i>y</i> <i>z</i> 


vậy Amax =


1 3


8 <i>x</i>  <i>y z</i> 2

<b>V</b>



<i><b>(1</b></i>
<i><b>điểm)</b></i>


<b>Câu V (</b>1 điểm)


Chứng minh rằng với mọi số tự nhiên <i>n</i> ( với <i>n</i> <sub> 2), ta có: ln</sub>2<i><sub>n </sub></i><sub> > </sub>


ln(<i>n</i>-1).ln(<i>n</i>+1)


 Với n = 2 thì BĐT cần chứng minh đúng


0.25



 Xét n > 2 khi đó ln(n – 1) > 0 BĐT tương đương với:




ln ln( 1)
ln( 1) ln


<i>n</i> <i>n</i>


<i>n</i> <i>n</i>





 <sub> (1) </sub>


0.25


 Hàm số f(x) =


ln
ln( 1)


<i>x</i>


<i>x</i> <sub> , với x > 2 là hàm nghịch biến, nên với n > </sub>


</div>
<span class='text_page_counter'>(4)</span><div class='page_container' data-page=4>

2 thì f(n) > f(n+1) 



ln ln( 1)
ln( 1) ln


<i>n</i> <i>n</i>


<i>n</i> <i>n</i>





 <sub>. BĐT (1) được chứng </sub>


minh.


<b> Câu V Câu V (1 điểm): Cho </b><i><b>x , y , z</b></i><b> là ba số thực thỏa mãn : </b>5<i>x</i><sub></sub> 5<i>y</i><sub></sub>5<i>z</i><sub></sub>1<b><sub> .Chứng</sub></b>


<b>minh r ng : ằ</b>




<i>x</i> <i>y</i> <i>z</i>


<i>x</i> <i>y z</i> <i>y</i> <i>z x</i> <i>z</i> <i>x y</i>


25 25 25


5 5  5 5  5 5 


   



<i>x</i> <i>y</i> <i>z</i>


5 5 5


4


 


<b>: </b>Đặt 5<i>x</i> <i>a</i>; 5<i>y</i> <i>b</i>; 5<i>z</i> <i>c</i><sub>. Từ giả thiết ta có: a, b, c > 0 và </sub><i>ab bc ca abc</i><sub></sub> <sub></sub> <sub></sub> <sub> </sub>


BĐT 


 


  


  


2 2 2


4


<i>a</i> <i>b</i> <i>c</i> <i>a b c</i>


<i>a bc b ca c ab</i> <sub> (*) </sub>


Ta có: (*) 


 



  


  


3 3 3


2 2 2 <sub>4</sub>


<i>a</i> <i>b</i> <i>c</i> <i>a b c</i>


<i>a</i> <i>abc b</i> <i>abc c</i> <i>abc</i>


 


  


     


3 3 3


( )( ) ( )( ) ( )( ) 4


<i>a</i> <i>b</i> <i>c</i> <i>a b c</i>


<i>a b a c</i> <i>b c b a</i> <i>c a c b</i>


Áp dụng BĐT Cơ-si, ta có:


 



  


 


3 <sub>3</sub>


( )( ) 8 8 4


<i>a</i> <i>a b a c</i> <i><sub>a</sub></i>


<i>a b a c</i> <sub> (1)</sub>


 


  


 


3 <sub>3</sub>


( )( ) 8 8 4


<i>b</i> <i>b c b a</i> <i><sub>b</sub></i>


<i>b c b a</i> <sub> ( 2) </sub>




 



  


 


3 <sub>3</sub>


( )( ) 8 8 4


<i>c</i> <i>c a c b</i> <i><sub>c</sub></i>


<i>c a c b</i> <sub> ( 3) </sub>


Cộng vế với vế các bất đẳng thức (1), (2), (3) suy ra điều phải chứng minh.


<b>Câu V</b> (1 điểm): Tìm giá trị nhỏ nhất của hàm số:


<i>x</i> <i>x</i> <i>x</i> <i>x</i>


<i>f x</i>


<i>x</i> <i>x</i>


4 3 2


2


4 8 8 5


( )



2 2


   




 


Tập xác định: D = R .


Ta có: <i>f x</i> <i>x</i> <i>x</i> <i>x</i> <i>x</i>


2


2


1


( ) 2 2 2


2 2


    


  ( BĐT Cô–si).


Dấu "=" xảy ra  <i>x</i>2–2<i>x</i>  2 1  <i>x</i>1<sub>.</sub>


Vậy: min f(x) = 2 đạt được khi x = 1.



<b>Câu V</b> (1 điểm): Cho các số thực không âm a, b. Chứng minh rằng:


<i>a</i>2 <i>b</i> 3 <i>b</i>2 <i>a</i> 3   2<i>a</i>  1 2<i>b</i> 1


4 4 2 2


       


      


       


       


Ta có: <i>a</i> <i>b</i> <i>a</i> <i>a</i> <i>b a</i> <i>a</i> <i>a b</i> <i>a b</i>


2


2 2 1 1 1 1


2 2 2 2


3 1


4 4


 


        



 


       


Tương tự: <i>b</i> <i>a</i> <i>a b</i>


2 1


2
3


4


    


. Ta sẽ chứng minh <i>a b</i> <i>a</i> <i>b</i>


2


1 <sub>2</sub> 1 <sub>(2</sub> 1


2 2 2


     


    


     



     


(*)


Thật vậy, (*)  <i>a</i> <i>b</i> <i>ab a b</i> <i>ab a b</i>


2 2 1 <sub>4</sub> 1


4 4


2 


       


 (<i>a b</i> )20.


Dấu "=" xảy ra  <i>a b</i>


1
2


 


</div>
<span class='text_page_counter'>(5)</span><div class='page_container' data-page=5>

Câu V. (1,0 điểm) Cho các số thực không âm <i>x , y , z</i> thoả mãn <i>x</i>2+<i>y</i>2+<i>z</i>2=3 . Tìm giá


trị lớn nhất của biu thc <i>A</i>=xy+yz+zx+ 5
<i>x</i>+<i>y</i>+<i>z</i> .


Đặt <i>t</i>=<i>x</i>+<i>y</i>+<i>z</i> <i></i> <i>t</i>2=3+2(xy+yz+zx)<i></i>xy+yz+zx=<i>t</i>



2


<i></i>3


2 .


Ta cã <sub>0</sub><i><sub>≤</sub></i><sub>xy</sub>+yz+zx<i>≤ x</i>2+<i>y</i>2+<i>z</i>2=3 nªn 3<i>≤t</i>2<i>≤</i>9<i>⇒</i>√3<i>≤t ≤3</i> vì <i>t></i>0.


Khi ú <i>A</i>=<i>t</i>


2<i><sub></sub></i><sub>3</sub>


2 +
5
<i>t</i> .


Xét hàm sè <i>f</i>(<i>t</i>)=<i>t</i>


2


2+
5


<i>t</i> <i>−</i>
3


2<i>,</i>√3<i>≤t ≤3 .</i>


Ta cã <i>f '</i>(<i>t</i>)=<i>t −</i>5



<i>t</i>2=
<i>t</i>3<i><sub>−5</sub></i>


<i>t</i>2 >0 v× <i>t ≥</i>√3 .


Suy ra <i>f</i>(<i>t</i>) đồng biến trên [<sub>√</sub>3<i>,3</i>] . Do đó <i>f</i>(<i>t</i>)<i>≤ f</i>(3)=14


3 .


Dấu đẳng thức xảy ra khi <i>t</i>=3<i>⇔x=y</i>=z=1.


VËy GTLN cđa <i>A</i> lµ 14


3 , đạt đợc khi <i>x=y=z</i>=1.


Câu V (1 điểm) Cho ba số <i>a, b, c </i>sao cho

{

<i>a , b , c</i>>0


abc=1 .


Tìm giá trị nhỏ nhất của biểu thức<i>A</i> = 1


<i>a</i>3(<i>b+c</i>)+


1


<i>b</i>3(<i>a</i>+c)+


1


<i>c</i>3(<i>b+a</i>)



Đặt <i> x</i> = 1


<i>a, y</i>=
1
<i>b, z</i>=


1


<i>c</i> . Khi đó:


<i>A</i>= <i>x</i>


3


1
<i>y</i>+


1
<i>z</i>


+ <i>y</i>


3


1
<i>x</i>+


1
<i>z</i>



+ <i>z</i>


3


1
<i>y</i>+


1
<i>x</i>


=¿ <i>x</i>3yz
<i>y</i>+<i>z</i>+


<i>y</i>3xz
<i>z</i>+<i>x</i> +


<i>z</i>3xy
<i>x</i>+<i>y</i> <i>≥</i>


3


2 (*)


Do abc=1<i>⇒</i>xyz=1 nªn ta cã <i>A</i>= <i>x</i>


2


<i>y</i>+<i>z</i>+
<i>y</i>2



<i>z</i>+<i>x</i>+
<i>z</i>2


<i>x</i>+<i>y</i> (1)


Ta chứng minh bất đẳng thức <i>a</i>+<i>b</i>+<i>c</i>


2


<i>a</i>2


<i>b</i>+<i>c</i>+
<i>b</i>2


<i>c</i>+<i>a</i>+
<i>c</i>2


<i>b</i>+<i>a</i>. ThËt vËy.


áp dụng bất đẳng thức Côsi cho các số dơng ta có:


<i>a</i>2
<i>b</i>+<i>c</i>+


<i>b</i>+<i>c</i>
4 <i>≥ a</i> ,


<i>b</i>2
<i>c</i>+<i>a</i>+



<i>c</i>+<i>a</i>
4 <i>≥ b</i> ,


<i>c</i>2
<i>a</i>+<i>b</i>+


<i>a</i>+<i>b</i>
4 <i>≥ c</i> .


Cộng ba bất đẳng thức cùng chiều trên ta có :


<i>a</i>+<i>b</i>+<i>c</i>
2


<i>a</i>2


<i>b</i>+<i>c</i>+
<i>b</i>2


<i>c</i>+<i>a</i>+
<i>c</i>2


<i>b</i>+<i>a</i>.


Bạn đọc tự đánh giá dấu “=” xảy ra khi<i> a</i> = <i>b</i> = <i>c.</i>


VËy <i>A= </i> <i>x</i>2


<i>y</i>+<i>z</i>+


<i>y</i>2


<i>z</i>+<i>x</i>+
<i>z</i>2


<i>x</i>+<i>y≥</i>


<i>x</i>+<i>y</i>+<i>z</i>


2 <i>≥</i>


3
2


3


√xyz=3
2


DÊu “=” x¶y ra khi <i>x</i> = <i>y</i> = <i>z </i>= 1. VËy min<i>A</i> = 3


2 khi <i>a </i>=<i> b</i> = c = 1 .


CâuV :( 1, 0 điểm). Cho x, y, z là ba số dương. Chứng minh bất đẳng thức sau :
3 2 3 2 3 2 2 2 2


2 y


2 x 2 z 1 1 1



x y + y z + z x x + y + z


CM bất đẳng thức 3 2 3 2 3 2 2 2 2


2 y


2 x 2 z 1 1 1


x y + y z + z x x + y + z <sub>với x > 0 ; y > 0 ; z > 0</sub>


+ Áp dụng bất đẳng thức Côsi cho hai số dương x3<sub> và y</sub>2<sub> ta có :</sub>


3 2 3 2


3 2


2 x 1


x y 2 x y 2 xxy


x y xy


    


</div>
<span class='text_page_counter'>(6)</span><div class='page_container' data-page=6>

Tương tự : 3 2


2 y 1


y z yz <sub>, dấu đẳng thức xảy ra khi và chỉ khi y</sub>3<sub> = z</sub>2



(2)


3 2


2 z 1


z x zx, dấu đẳng thức xảy ra khi và chỉ khi z3 = x2


(3)


+ Áp dụng BĐT(dễ CM ) ab bc ca a   2b2c2<sub>(dấu đẳng thức xảy ra khi và chỉ khi </sub>


a = b = c )


ta có : 2 2 2


1 1 1 1 1 1


xy + yz+ zx x + y + z <sub> , dấu đẳng thức xảy ra khi và chỉ khi x = y = z (4)</sub>


+ Từ (1), (2), (3) và (4) ta có BĐT cần C/minh . Dấu đẳng thức xảy ra khi và chỉ khi
x = y = z > 0


b/.Cho a, b, c>0; abc=1 . Ch ng minh r ng ư ă


3 3 3 <sub>3</sub>


(1 )(1 ) (1 )(1 ) (1 )(1 ) 4


<i>a</i> <i>b</i> <i>c</i>



<i>b</i> <i>c</i>  <i>c</i> <i>a</i>  <i>a</i> <i>b</i> 


      <sub>.</sub>


Ap d ng b t u â đăng th c côsi cho ba s , ta coư ô


 


  


 


3 <sub>1</sub> <sub>1</sub> <sub>3</sub>


(1 )(1 ) 8 8 4


<i>a</i> <i>c</i> <i>b</i> <i>a</i>


<i>b</i> <i>c</i>


 


  


 


3 <sub>1</sub> <sub>1</sub> <sub>3</sub>


(1 )(1 ) 8 8 4



<i>b</i> <i>c</i> <i>a</i> <i>b</i>


<i>c</i> <i>a</i>




 


  


 


    


3 <sub>1</sub> <sub>1</sub> <sub>3</sub>


(1 )(1 ) 8 8 4


3 1
(1)


4 2


<i>c</i> <i>a</i> <i>b</i> <i>c</i>


<i>a</i> <i>b</i>


<i>VT</i> <i>a b c</i>



D u b ng x y ra khi â ă a


1 1 1


1


8 8 8


1


<i>a</i> <i>c</i> <i>b</i>


<i>a b c</i>
<i>abc</i>


   


 




   




 <sub></sub>


 <sub>. </sub>


V y â



3 3 3


(1) (1)


2 4 4


<i>VT</i>   <i>VT</i>  


pcm.


đ


Câu 5: <i>( 1 điểm)</i> Cho <i>a b c</i>, , là các số thực không âm, đôi một khác nhau. Chứng minh rằng:
2


<i>ab</i> <i>bc</i> <i>ca</i> <i>a b c</i>


<i>a b b c c a</i>


 


  


   <sub> Hỏi dấu “=” xảy ra khi nào?</sub>
áp dụng bất đẳng thức Côsi cho các số <i>a b c</i>, , ta đợc:




 



 



 



2 1


2
2


2 2


2
2


2 3


2
2


<i>ab</i> <i>ab</i> <i>ab</i> <i>ab</i>


<i>a b</i> <i>ab</i>


<i>a b</i> <i>ab</i> <i>a b</i>


<i>bc</i> <i>bc</i> <i>bc</i> <i>bc</i>


<i>b c</i> <i>bc</i>


<i>b c</i> <i>bc</i> <i>b c</i>



<i>ca</i> <i>ca</i> <i>ca</i> <i>ca</i>


<i>c a</i> <i>ca</i>


<i>c a</i> <i>ca</i> <i>c a</i>


     


 


     


 


     


 


</div>
<span class='text_page_counter'>(7)</span><div class='page_container' data-page=7>



2
1


2 2 2 2


2


<i>ab</i> <i>bc</i> <i>ca</i> <i>ab</i> <i>bc</i> <i>ca</i>



<i>a b b c c a</i>


<i>ab</i> <i>bc</i> <i>ca</i> <i>a b b c c a</i>


<i>a b b c c a</i>


<i>ab</i> <i>bc</i> <i>ca</i> <i>a b c</i>


<i>dpcm</i>
<i>a b b c c a</i>


 


  


  


  


 


    <sub></sub>   <sub></sub>


    


 


    


  



DÊu “=” x¶y ra khi vµ chØ khi <i>a b c</i>  <sub>.</sub>


2). Cho x, y, z là ba số thực dương thay đổi và thỏa mãn: <i>x</i>2+<i>y</i>2+<i>z</i>2<i>≤</i>xyz . Hãy tìm giá trị


lớn nhất của biểu thức: <i>P</i>= <i>x</i>
<i>x</i>2+yz+


<i>y</i>
<i>y</i>2+zx+


<i>z</i>
<i>z</i>2+xy .


Vì <i>x ; y ; z</i>>0 , Áp dụng BĐT Cơsi ta có: <i>P≤</i> <i>x</i>


2

<sub>√</sub>

<i>x</i>2<sub>yz</sub>+


<i>y</i>


2

<sub>√</sub>

<i>y</i>2<sub>zx</sub>+


<i>z</i>


2

<sub>√</sub>

<i>z</i>2<sub>xy</sub> =




¿1



4

(


2


√yz+
2


√zx+
2


√xy

)



1


4

(


1


<i>y</i>+


1


<i>z</i>+


1


<i>z</i>+


1


<i>x</i>+



1


<i>x</i>+


1


<i>y</i>

)

=


1
2

(



yz+zx+xy


xyz

)

<i>≤</i>
1
2

(



<i>x</i>2


+<i>y</i>2+<i>z</i>2


xyz

)



1
2

(



xyz
xyz

)

=


1


2


Dấu bằng xảy ra <i>x=y</i>=z=3 . Vy MaxP = 1<sub>2</sub>


Câu V <i>(1 điểm)</i> Cho x, y, z là 3 số thực dơng thỏa m·n xyz=1. Chøng minh r»ng




1 1 1


1


1 1 1


<i>x y</i>  <i>y z</i> <i>z x</i>


Đặt x=a3<sub> y=b</sub>3<sub> z=c</sub>3<sub> thì x, y, z >0 vµ abc=1.Ta cã</sub>


a3<sub> + b</sub>3<sub>=(a+b)(a</sub>2<sub>+b</sub>2<sub>-ab)</sub><sub></sub><sub>(a+b)ab, do a+b>0 vµ a</sub>2<sub>+b</sub>2<sub>-ab</sub><sub></sub><sub>ab</sub>


 <sub> a</sub>3<sub> + b</sub>3<sub>+1</sub><sub></sub><sub> (a+b)ab+abc=ab(a+b+c)>0</sub>


 3 3



1 1


a  b 1 ab a b c  


Tương tự ta có





3 3


1 1


c 1 bc a b c


<i>b</i>     


,



3 3


1 1


a 1 ca a b c


<i>c</i>     


Céng theo vÕ ta cã


1 1 1


1 1 1


<i>x y</i>   <i>y z</i>   <i>z x</i>  <sub>=</sub> 3 3


1



a  b 1<sub>+</sub> 3 3
1
c 1


<i>b</i>   <sub>+</sub> 3 3


1
a 1


<i>c</i>  




1 1 1 1


a b c <i>ab bc ca</i>


 


 


 


   <sub>=</sub>



1


1
a b c  <i>c a b</i>  



DÊu b»ng x¶y ra khi x=y=z=1


</div>
<span class='text_page_counter'>(8)</span><div class='page_container' data-page=8>

Tìm giá trị nhỏ nhất của biểu thức P = log22<i>x</i> 1 log22 <i>y</i> 1 l g<i>o</i> 22 <i>z</i>4


trong đó x, y, z là các số dương thoả mãn đièu kiện xyz = 8


Theo bất đẳng thức Minkowski: <i>a</i>12<i>b</i>12  <i>a</i>22<i>b</i>22  <i>a</i>32<i>b</i>32  (<i>a</i>1<i>a</i>2<i>a</i>3)2(<i>b</i>1<i>b</i>2<i>b</i>3)2


Dấu đẳng thức xảy ra khi và chỉ khi:


3
1 2
1 2 3


<i>a</i>


<i>a</i> <i>a</i>


<i>b</i> <i>b</i> <i>b</i>


Ta có P  log (22 <i>xyz</i>) 4 2 = 5 ( vì xyz = 8)


Vậy minP = 5 khi và chỉ khi


2 2 2 2


log log log log ( ) 3


1 1 2 4 4



<i>x</i> <i>y</i> <i>z</i> <i>xyz</i>


    <i><sub>x</sub></i> <i><sub>y</sub></i> 4<sub>8;</sub><i><sub>z</sub></i> <sub>2 2</sub>


   


<i>Câu V (1 điểm). Tìm giá trị nhỏ nhất của biểu thức </i> 2 2


2
2 <sub>4</sub> <sub>6</sub>
4
<i>y</i>
<i>x</i>
<i>xy</i>
<i>x</i>
<i>y</i>
<i>P</i>





Ta có 5 5


)
3
(
)
(
5


)
9
6
(
2
2
2
2
2
2
2
2
2












<i>y</i>
<i>x</i>
<i>y</i>
<i>x</i>
<i>y</i>

<i>x</i>
<i>y</i>
<i>x</i>
<i>y</i>
<i>xy</i>
<i>x</i>
<i>P</i>
5



 <i>MinP</i> <sub> khi </sub>














0
3
0
0
3


2
2 <i><sub>y</sub></i>
<i>y</i>
<i>x</i>
<i>y</i>
<i>x</i>
<i>y</i>
<i>x</i>


2, Cho các số thực dơng a, b, c tho¶ m·n ab+bc+ca=abc . Chøng minh r»ng:


<i>a</i>4


+<i>b</i>4
ab(<i>a</i>3+<i>b</i>3)+


<i>b</i>4


+<i>c</i>4
bc(<i>b</i>3+<i>c</i>3)+


<i>c</i>4


+<i>a</i>4
ca(<i>c</i>3+<i>a</i>3)<i>≥</i>1


Tõ <i>a</i>4


+<i>b</i>4<i>≥ a</i>3<i>b</i>+ab3<i>⇒</i>2(<i>a</i>4+<i>b</i>4)<i>≥ a</i>4+<i>a</i>3<i>b</i>+<i>b</i>4+ab3=(<i>a</i>3+<i>b</i>3)(<i>a</i>+<i>b</i>) .



VËy <i>a</i>


4


+<i>b</i>4
ab(<i>a</i>3+<i>b</i>3) <i>≥</i>


<i>a</i>+<i>b</i>
2ab=
1
2

(


1
<i>a</i>+
1
<i>b</i>

)

.


Tơng tự cho các bất đẳng thức còn lại, suy ra đpcm.


Câu V: ( 1,0 điểm)


Tìm các giá trị của tham số <i>m</i> để phương trình sau có nghiệm duy nhất thuộc đoạn


[

<i>−</i>1


2<i>;</i>1

]

: 3

1<i>− x</i>2<i>−</i>2

<i>x</i>3+2<i>x</i>2+1=<i>m</i> ( <i>m∈R</i> ).


Đặt <i>f x</i>

 

3 1 <i>x</i>2  2 <i>x</i>32<i>x</i>21, suy ra <i>f x</i>

 

xác định và liên tục trênđoạn 1 12;


 


 
 <sub>.</sub>

 


'
2


2 3 2 2 3 2


3 3 4 3 3 4


1 2 1 1 2 1


<i>x</i> <i>x</i> <i>x</i> <i>x</i>


<i>f x</i> <i>x</i>


<i>x</i> <i>x</i> <i>x</i> <i>x</i> <i>x</i> <i>x</i>


 
 
   <sub></sub>  <sub></sub>
       <sub>.</sub>
;
1 1
2


<i>x</i>  


  <sub></sub> <sub></sub>



  <sub> ta có </sub> 2 3 2


4 <sub>3</sub> <sub>4 0</sub> 3 3 4 <sub>0</sub>


3 <sub>1</sub> <sub>2</sub> <sub>1</sub>


<i>x</i>


<i>x</i> <i>x</i>


<i>x</i> <i>x</i> <i>x</i>




       


   <sub>.</sub>


Vậy: <i>f x</i>'

 

 0 <i>x</i>0.
Bảng biến thiên:


 



 



' || ||


1 <sub>0</sub> <sub>1</sub>


2



0
1


3 3 22


</div>
<span class='text_page_counter'>(9)</span><div class='page_container' data-page=9>

Dựa vào bảng biến thiên, ta có: Phương trình đã cho có 1 nghiệm duy nhất thuộc 1 12;


 




 


 


3 3 22


4


2


<i>m</i> 


   


</div>

<!--links-->

Tài liệu bạn tìm kiếm đã sẵn sàng tải về

Tải bản đầy đủ ngay
×